LSAT and Law School Admissions Forum

Get expert LSAT preparation and law school admissions advice from PowerScore Test Preparation.

 Administrator
PowerScore Staff
  • PowerScore Staff
  • Posts: 8919
  • Joined: Feb 02, 2011
|
#85375
Complete Question Explanation

Strengthen. The correct answer choice is (D).

Answer choice (A):

Answer choice (B):

Answer choice (C):

Answer choice (D): This is the correct answer choice.

Answer choice (E):

This explanation is still in progress. Please post any questions below!
 parytownson
  • Posts: 11
  • Joined: Feb 12, 2021
|
#88715
Hello Powerscore Team,

I have two questions:

1.) Why is this problem not categorized as "Justify the Conclusion?"

2.) Is this the proper way to diagram this problem:

Premise: Classical Account must be discarded (~CA)

Conclusion: Sound change theory in general should be discarded (~SC)

Answer choice (D): "If it is a theory of sound change, then it relies heavily on the classical account" (SC :arrow: CA). If I then invoke the contraposition of this conditional (~CA :arrow: ~ SC) then the argument's conclusion can be justified. [/list][/list]
 Robert Carroll
PowerScore Staff
  • PowerScore Staff
  • Posts: 1787
  • Joined: Dec 06, 2013
|
#88767
pary,

As far as classification goes, this post by Dave explains why we call this a Strengthen: viewtopic.php?p=9480#p9480

"does most to justify" is the wording in this particular question that makes this less demanding than a Justify, thus only a Strengthen question.

The conditional diagramming works, although I don't think it's needed. But you're right - if all theories rely on the classical theory, and a central tenet of that must be discarded, than all the things that rely on the classical theory are in danger.

Robert Carroll

Get the most out of your LSAT Prep Plus subscription.

Analyze and track your performance with our Testing and Analytics Package.